Udowodnij indukcyjnie.

Ze względu na specyfikę metody - osobny dział.
Madzzia
Użytkownik
Użytkownik
Posty: 66
Rejestracja: 12 mar 2021, o 12:54
Płeć: Kobieta
wiek: 18
Podziękował: 21 razy

Udowodnij indukcyjnie.

Post autor: Madzzia »

Udowodnij indukcyjnie, że dla \(\displaystyle{ n}\) naturalnych:
\(\displaystyle{ \sum_{k=2}^{n} \frac{1}{k^2} < 1 }\)

Rozpoczęłam to zadanie tak:

1) Krok bazowy:
Dla \(\displaystyle{ n=2}\)
\(\displaystyle{ L= \frac{1}{4} < 1}\)

2) Udowodnię, że jeżeli \(\displaystyle{ \sum_{k=2}^{n} \frac{1}{k^2} < 1 }\) to \(\displaystyle{ \sum_{k=2}^{n+1} \frac{1}{k^2} < 1 }\)

\(\displaystyle{ \sum_{k=2}^{n+1} \frac{1}{k^2} = \sum_{k=2}^{n} \frac{1}{k^2}+ \frac{1}{(n+1)^{2}} }\)

Jak dokończyć ten dowód?
Tmkk
Użytkownik
Użytkownik
Posty: 1718
Rejestracja: 15 wrz 2010, o 15:36
Płeć: Mężczyzna
Lokalizacja: Ostrołęka
Podziękował: 59 razy
Pomógł: 501 razy

Re: Udowodnij indukcyjnie.

Post autor: Tmkk »

Tak Ci się nie uda. Pewnie zauważyłas w czym jest problem - jeśli teraz użyjesz założenia indukcyjnego to będziesz miała \(\displaystyle{ 1 + (\hbox{coś dodatniego}) < 1}\) i bezsensu.

Aby indukcja (tak bezpośrednio) zadziałała, musisz trochę wzmocnić tezę, czyli zamiast pokazywać, że \(\displaystyle{ \sum_{k=2}^n\frac{1}{k^2}< 1}\), pokaż coś więcej, czyli że \(\displaystyle{ \sum_{k=2}^n \frac{1}{k^2} < 1 - (\hbox{coś zależnego od }n)}\).

A czym ma być to coś zależnego od \(\displaystyle{ n}\)? To trzeba sobie pokombinować i wymyślić (lub znać inny dowód tej nierówności, bez indukcji). Jeśli będziesz miała problem, daj znać, to podam, co tam można sobie wpisać i wtedy indukcja zadziała.
Madzzia
Użytkownik
Użytkownik
Posty: 66
Rejestracja: 12 mar 2021, o 12:54
Płeć: Kobieta
wiek: 18
Podziękował: 21 razy

Re: Udowodnij indukcyjnie.

Post autor: Madzzia »

Okej zatem udowodnię, że:
\(\displaystyle{ \sum_{k=2}^{n} \frac{1}{k^{2}} < 1- \frac{1}{n} }\)

1) Krok bazowy:
dla \(\displaystyle{ n = 2}\)
\(\displaystyle{ \frac{1}{4}< \frac{1}{2} }\)
Prawda.

2) Pokażę, że jeżeli \(\displaystyle{ \sum_{k=2}^{n} \frac{1}{k^{2}} < 1- \frac{1}{n} }\) to \(\displaystyle{ \sum_{k=2}^{n+1} \frac{1}{k^{2}} < 1- \frac{1}{n+1} }\)

\(\displaystyle{ \sum_{k=2}^{n+1} \frac{1}{k^{2}} =\sum_{k=2}^{n} \frac{1}{k^{2}}+ \frac{1}{(n+1)^{2}} }\)

Czyli wystarczy pokazać, że
\(\displaystyle{ 1- \frac{1}{n+1}- \frac{1}{(n+1)^{2}} \ge 1- \frac{1}{n} }\)
\(\displaystyle{ \frac{n^{2}+2n+1-n-1-1}{(n+1)^{2}} \ge \frac{n-1}{n} }\)

\(\displaystyle{ n(n^{2}+n-1) \ge (n-1)(n^{2}+2n+1)}\)

\(\displaystyle{ n^{3}+n^{2}-n \ge n^{3}+2n^{2}+n-n^{2}-2n-1}\)

\(\displaystyle{ 0 \ge -1}\)
To jest prawdziwe zatem tym bardziej prawdziwe jest zdanie:
\(\displaystyle{ \sum_{k=2}^{n} \frac{1}{k^{2}} < 1 }\)

Czy to jest okej? :roll:
Tmkk
Użytkownik
Użytkownik
Posty: 1718
Rejestracja: 15 wrz 2010, o 15:36
Płeć: Mężczyzna
Lokalizacja: Ostrołęka
Podziękował: 59 razy
Pomógł: 501 razy

Re: Udowodnij indukcyjnie.

Post autor: Tmkk »

Tak, dokładnie o to chodziło.

Jeśli chciałabyś mieć trochę mniej rachunków, to zobacz, że warunek, który sprawdzasz, czyli \(\displaystyle{ 1 - \frac{1}{n+1} - \frac{1}{(n+1)^2} \ge 1 - \frac{1}{n}}\) jest równoważny takiemu (te jedynki aż prosi się skrócić)

\(\displaystyle{ \frac{1}{(n+1)^2} \le \frac{1}{n} - \frac{1}{n+1} = \frac{1}{n(n+1)}}\),

co redukuje się do sprawdzenia \(\displaystyle{ (n+1)^2 \ge n(n+1)}\), a to jest oczywiste.
ODPOWIEDZ